LSAT and Law School Admissions Forum

Get expert LSAT preparation and law school admissions advice from PowerScore Test Preparation.

 Administrator
PowerScore Staff
  • PowerScore Staff
  • Posts: 8917
  • Joined: Feb 02, 2011
|
#38362
Complete Question Explanation
(The complete setup for this game can be found here: lsat/viewtopic.php?t=15015)

The correct answer choice is (E)

This Could Be True, Except question asks you to identify which variable cannot be third. If you are like most test-takers who solved this game, you did not have the requisite Not-Law on the third position. This would be unnerving given the prospect of trying out each and every answer choice. There is (almost always) an easier way to proceed.

Whenever you stumble upon a Global, Cannot Be True question that cannot be answered by referring to your initial set of Not Laws, return to the List question and see what could be true. Here, the correct answer to Question #17 was answer choice (C), in which M was third. Therefore, we can eliminate answer choice (A) in Question #20. Next, examine the local diagrams already made. For instance, our solution for Question #18 shows that R could be third (local setup 2.2.), helping us eliminate answer choice (B) in Question #20.

We are now left with answer choices (C), (D), and (E). One approach would be to test each of these three possibilities on the third position: two will work, and one will not (this will be the correct answer choice). A superior strategy, however, would be to skip this question entirely (for now) and tackle a few more local questions in the hopes of creating additional local diagrams that show what other variables could be third. And indeed, the local solution to Question #23 shows that T could be third, further eliminating answer choice (D).

With only two contenders left, creating the local diagrams will not be terribly time-consuming:

Answer choice (C): If S were third, then V would be fourth in compliance with the last rule of the game. Consequently, P would be second, because S and P must be consecutive. Thus:
PT71_D13 LG Explanations_Game #4_#20_diagram 1.png
Next, we need to accommodate the R _ M split-block, which must be positioned in spaces 5—7. The two remaining positions—the first and the sixth—will be occupied by T and W, respectively, in compliance with the first rule:
PT71_D13 LG Explanations_Game #4_#20_diagram 2.png
Our local diagram proves that S could be third, leaving answer choice (E) as the only possible contender. Therefore, W cannot be third, and answer choice (E) must be correct. Make sure to add this Not Law to your main diagram, and use the inference in subsequent questions, if necessary.

Let’s take a moment to more fully understand why W cannot be third. If W were third, then V would be fourth, according to the fourth rule. Meanwhile, T would be either first or second, because of the sequencing restriction placed by the first rule. We arrive at the following setup:
PT71_D13 LG Explanations_Game #4_#20_diagram 3.png
This setup cannot accommodate the two remaining blocks (the R _ M block along with the rotating PS block), proving that W cannot be third.

Of course, you would not need to do any of this if you had access to the game’s templates, which quickly show that each of M, R, S, and T can occupy the third position. W, however, is not third on any of the templates, proving answer choice (E) correct.
You do not have the required permissions to view the files attached to this post.

Get the most out of your LSAT Prep Plus subscription.

Analyze and track your performance with our Testing and Analytics Package.